LSAT and Law School Admissions Forum

Get expert LSAT preparation and law school admissions advice from PowerScore Test Preparation.

 Administrator
PowerScore Staff
  • PowerScore Staff
  • Posts: 8916
  • Joined: Feb 02, 2011
|
#26993
Complete Question Explanation

Justify the Conclusion. The correct answer choice is (A)

The conclusion in this Justify question is that all genetic mutation is random. This comes from the idea that in specific experiments (with certain conditions) genetic mutations occurred at random. To prove such a strong conclusion you need a very broad, all-encompassing answer choice showing that if even one genetic mutation is random (which you know from the experiments is the case) then all genetic mutation must be random (the conclusion).

Answer choice (A): This is the correct answer choice. Since you know that some genetic mutations are random (the experiments), and this answer says essentially all are or none are, then it must be the case that all are, and the conclusion is proven correct.

Answer choice (B): This answer does not address mutations so it cannot be correct.

Answer choice (C): This answer choice assumes that the conclusion is true (“if all are random”), but does not provide information to prove that the conclusion is true.

Answer choice (D): This does not provide information that would show that from the experiments all genetic mutation must be random, so it is incorrect.

Answer choice (E): This answer choice also does not address genetic mutations, so it cannot be correct.
 mford
  • Posts: 25
  • Joined: Aug 27, 2011
|
#1636
The mechanistic approach is somewhat difficult to understand from an 'in-practice' perspective. For instance, I have trouble understanding why the answer to #8 on page 4-21 of the full course book is A. Any tips?
User avatar
 Dave Killoran
PowerScore Staff
  • PowerScore Staff
  • Posts: 5853
  • Joined: Mar 25, 2011
|
#1663
Sure, I'll try to give you a hand.

First, you are looking at a question that is widely considered quite difficult, and it contains an idea that throws a lot of people off at first (and this idea makes the Mechanistic Approach harder to apply; this isn't the greatest question to display the powers of the technique, although it will help us narrow it down to two answers).

Here's how the argument sets up in very broad terms:

  • Premise: (some) genetic mutations in bacteria occurred at random.

    Conclusion: all genetic mutation is random.
Second, from a mechanistic standpoint, the biggest change from premise to conclusion is the "some" and "all" elements. But, the conclusion contains the "all" element, so we need a way to get to that conclusion in the answer choices. Interestingly, only answer choices (A) and (C) reference the "all" element, so those two are the most attractive Contenders from the outset. Let's examine each answer more closely.

Answer choice (A): This is the correct answer. If we add this statement to the stimulus, we see an interesting structure. I'm going to remove the "genetic mutation" portion because that's common to all elements, and removing it allows us to isolate the numbers game being played:

  • (A): it's all or none.
    Premise: it's some.
    Conclusion: Well, since it's all or none, and the presence of some rules out none, it must be all.

That's literally the process at work here, and this answer choice allows us to connect the thread from "some" to "all" in the stimulus. However, and this is what makes this question a classic, they've made this connection in a very tricky fashion.

Answer choice (C): This answer choice can be diagrammed as:

  • All random in bacteria :arrow: all random in life
The problem here is that the sufficient condition, "All random in bacteria," isn't know (just "some" is known, which while it could be "all," isn't known to be "all" for sure) so adding this statement to the premises doesn't give us "all random in life" as the conclusion.

This answer can be analogized as follows:

  • (C): If all high school football players are athletic, then football players at every level of the sport are athletic
    Premise: Some high school football players are athletic.
    Conclusion: No conclusion can be drawn.
Does that help with that problem? Please let me know. It's a tough one, but ultimately one of the best Justify questions out there for understanding how the mindset of the test makers works.

Thanks!
 mford
  • Posts: 25
  • Joined: Aug 27, 2011
|
#1696
Yes some includes all; very sly. So the test makers introduce a shell game element by claiming in choice C that "because all bacteria have random genetic mutations...." i.e. making a different argument than the one presented in the stimulus. Genius!
User avatar
 Dave Killoran
PowerScore Staff
  • PowerScore Staff
  • Posts: 5853
  • Joined: Mar 25, 2011
|
#1700
Exactly. (C) is a brilliant answer because it contains the element you want to produce, but it contains it in a manner that doesn't allow you to produce that result.

Make sure you understand this question perfectly—it is one of my all-time favorites for understanding how the whole Justify idea works on this test!
 taylor
  • Posts: 7
  • Joined: Jun 03, 2012
|
#5467
Could someone please explain what makes "A" the correct answer choice? I don't understand how setting up a binary (all mutation being random vs. none being random) can make the argument's conclusion airtight. Thanks in advance!
User avatar
 Dave Killoran
PowerScore Staff
  • PowerScore Staff
  • Posts: 5853
  • Joined: Mar 25, 2011
|
#5468
Hi Taylor,

You've run across one of the landmark LSAT questions. Let's first look at the basic structure of the argument (just the relevant parts):
  • Premise: Some genetic mutation is random.

    Conclusion: All genetic mutation is random.
The question stem is a Justify question, and the correct answer very cleverly uses the "some" portion from the stimulus. Let's consider this from the perspective of the Justify Formula:
  • Premise: Some genetic mutation is random.

    Answer choice (A): Either all genetic mutation is random or none is.
The answer stipulates that all is random, or none is. But, from the premise, we know that some mutation is random, so the possibility of none being random is eliminated. Thus, by adding answer choice (A) to the premise, the following conclusion results:
  • Conclusion: All genetic mutation is random.
As that is the same conclusion in the stimulus, (A) justifies the conclusion and is the correct answer.

Please let me know if that helps. Thanks!
 taylor
  • Posts: 7
  • Joined: Jun 03, 2012
|
#5516
Thanks, Dave! Your explanation makes a lot of sense.
 SMR
  • Posts: 16
  • Joined: Mar 21, 2014
|
#14459
Hello, can someone please tell me if the technique used to solve this question is the mechanistic approach? If so, can you please explain how exactly it is used? Thank you!
 Nikki Siclunov
PowerScore Staff
  • PowerScore Staff
  • Posts: 1362
  • Joined: Aug 02, 2011
|
#14460
Hi SMR,

At the most basic level, the argument can be diagrammed as follows:
  • Premise: Genetic Mutation :some: Random

    Conclusion: Genetic Mutation :arrow: Random
In other words, the author observes that in some experiments, genetic mutations occur at random. She extrapolates from this that all genetic mutations are random. The question stem asks you to prove that conclusion.

Answer choice (A) establishes that only two options are possible: either all mutations are random, or else none are random. The second option is ruled out by the results of the experiments, because some mutations have been shown to be random. So, if it's "all or none," and "none" is not an option, then it must be "all." This is a fairly mechanistic way of looking at this argument, though some of the tips on page 4-28 cannot be applied here. This is partly because the argument in Question 8 is not purely conditional (as is the argument in Question 7, for instance). Nevertheless, it shows how the correct answer choice in Justify questions must prove the conclusion 100%. Some of the other answer choices - such as (B), (D), and (E) - provide support for the reliability of the experiments conducted, but do not prove the conclusion.

Hope this helps! Let me know.

Get the most out of your LSAT Prep Plus subscription.

Analyze and track your performance with our Testing and Analytics Package.